You are on page 1of 41
INDIAN STATISTICAL INSTITUTE Mid-Semestral Examination Second semester M, Stat- First year 2015 Stochastic Processes Date: 7th September, 2015 Maximum Marks: 40 Duration: 2 hours 30mins Anybody caught using unfair means will immediately get 0. Please try to explain every step. Only class notes are allowed in the exam. (1) Consider the Poisson process N = ys1 €), on (—c0, <0] with points {je}i>1 and mean measure je(dx) = e~*dx. Let Yi = sup jx be the largest point and ¥ be the second largest point. (a) Compute the distribution of Y;. (b) Compute the joint distribution of ¥1, Ya. [4+4 points] (2) Vehicles pass a crossing at the instants of a Poisson process of intensity A; you need a gap length of at least @ in order to cross. Let T be the first time at which you could succeed in crossing to the other side. (a) Show that E(T] = =; (b) Find E[e®"] for 6 > 0; (c) Suppose there are 2 lanes to cross, carrying independent Poissonian traffic with respective rates A and x. Find the expected time to cross in two cases when (i) there is an island between 2 lanes and (ii) you must ctoss both in one go. Which one is higher? [44242 points] (3) Let Z, be the size of the m-th generation of a branching process with Zo = 1 and P(Z; = k) = 2-* fork 2 0. Show directly that as 7 —> 0%, P(Zn < 2ytt|Zn > 0) + 1- efor y > 0. [8 points] (4) Consider a branching process with generation sizes Zy satisfying Zo = 1, and P(Z; = 0) = 0. Pick two individuals at random (with replacement) from the n-th generation and let L be the index of the generation which contains their most recent ancestor. (a) Show that P(L =r) = E[Z;1] — E[Z;},] for0 0? [6+2 points] (5) We know that extinction probability q = lim, P(Zn = 0) = limy-s00 Pn (0). If po > 0, then show that for every s € 0,1), @n(s) > q as n + 00, [8 points] (6) Let N be a Poisson process in IR? with constant intensity A, and let Ray < Rg) < +++ be the ordered distances from the origin of the points of the process, (a) Show that Rhy Roy +++ are points of a Poisson process on (0,00) with. intensity Az. (b) Show that Rq) has a density function [4#4 points] INDIAN STATISTICAL INSTITUTE MID-SEMESTRAL EXAMINATION M Stat (1* Year) (B-Stream) 2015-16 Subject: Statistical Inference Full Marks: 40 Date: 09.09.2015 Duration: 1 hr. 40 mins, Attempt all Questions a) Define admissible decision rules and complete class of decision rules. b) Show that if minimal complete class of decision rules exists, then it is the set ofall admissible rules. ©) Show by an example that the set of admissible rules may be non-empty, but minimal complete class may not exist. d) By an example show that intersection of two complete classes may not be complete. e) Let Xi, Xo, -- eee , Xq (n>1) be a sample of independent and identically distributed observations from N (0 , 02). Show that MLE for o? may not be admissible under square error loss. [245434545] a) Let for © = @ = R we have the following loss function: L@a)=%lO-al if | Osa =%|O-al if Oza Let p (8/x) be the posterior density of 6 when x is observed. Derive the Bayes decision rule in terms of some t** quantile of p (0/x). Find t b) Let © = {©1, Or} and the risk set ({.e the set of risk functions as subset of R2) be given by {(x1,x2) : Ga - 3)? + (2-5)? 3} Derive the minimax ruleie the point denoting minimax rule. ©) Fora statistical decision problem the following are given O=a=[0,~), L(0,a) = |@-al foroeO and aed £(x/0)= 1/8, 00 Show that G? is well defined under the mull and G? > 0 with equality if and only if ¢j = nyo for all j (b) For observed {7} define the divergence measure 2 DOV= sea Le lles/mnso)* — 1] Show that lim D(A) = G?. (12+8=20] 2a) For I x J contingency tables, explain why the variables are independent when the (I~ 1) x (J ~ 1) differences yy ~ my = 0) § = 142-0 — 1), and 5 = 1,2,.-(J-D- 7 (b) For a diagnostic test of a certain disease, let m denote the probability that the diagnosis is positive given that the subject has the disease, and let m2 denote the probability that the diagnosis is positive given that the subject does not have the Mieane, Let p denote the probability that a subject has the disease, Find the probability that the subject has the disease with positive diagnosis 4 [u5+10 = P.T.O. 3 (a) Describe the Fisher’s exact test to test the hypothesis of no association in a2 contingency table. When a test statistic has continuous distribution, the P-y has an uniform distribution under the null. For Fisher's exact. test. explain Pr(P — value < a) < @ (b) Suggest a suitable experimental design in a 2% 2 table which produces an unbi estimate of the odds ratio. (c) Consider a 2x2 contiongency table of joint distribution of two binary random: able X and Y. Let (X1, Yi), (X2,¥,),.-- (Xn ¥a) ben independent and identi distributed replications of (X,Y). Find the conditional distribution of D ) given D,X; =r and SD, ¥j = for suitable for different choices of nonnege integers (r,1) [lo + 5+ 10: INDIAN STATISTICAL INSTITUTE, Mid-semester Examination ; Semester I (2015-2016) M. Stat Ist Year Multivariate Analysis Date: 14. 9. 15 Maximum marks: 60 Note: Answer all questions. Maximum you an score is 60 Suppose X is an n x p data matrix from N,(jz,B) where © = o°((1-pil-+ pS], J is apxp matrix with all entries equal to one. It is known that o > 0 and pny 0] < E[X?]/E[X] for any random variable X taking non- hegative values. (b) Let Z, be the size of the n-th generation of a branching process with Zp = 1 and P[Z, = k) = gp* for k > 0 where p > 1/2. Use (a) to show that E[Zn/u"lZn > 0} < 2p/(p — q) where = p/q. Also show that E[2#|Z, > 0) + 2 as n > 00. [34+7=10) Consider a pure renewal process N(t). Denote U(¢) = ELV (t)]- ao (a) Show V(t) := ELN(?] = D2 (2n - FO (2) (b) Find the Laplace transform of 3, nF" ?*(t). Use this to write V(t) in terms of U(t) and derive the renewal equation. (c) Find an expression for V only in terms of U. (a) What is V(t) if V is a Poisson process? [34-34242=10] 6. Consider a renewal process NV and suppose that arrival is ‘overlooked’ with probability q, independently of all other arrivals. Let M(t) be the number of arrivals which are 4. deleted upto time t/p where p = 1 (a) Show that M is a renewal process whose inter-arrival time distribution function Gy is given by G(2) = Sopa! F,(«/p) = where F, is the distribution function of the time of the n-th arrival in the original process N. (b) Find the characteristic function of G, in terms of F and show that lim). 9 Gp(s) = 1-e**!# for s > 0, so long as inter-arrival times in the original process have finite mean p. [5+5=10} Indian Statistical Institute M.Stat. First year, First Semestral Exam: 2015-16 + Regression Techniques DU 19+1+/S- — Maximum Marks: 60, Duration: 3 hours Answer all questions. Show your work to get full credit. Marks will be deducted for untidiness and bad handwriting. 1. (a) Under the multiple regression model ¥ = XJ + where © ~ N(0,02A), A is positive definite, how will you test the hypothesis Hy: C3 = ¢, for some q x p matrix C with rank y? (1) In a biomedical study, data are collected on total medical expenditure and related covariates for a particular season of the year from n individuals. In the data, m(< nt) responses are found to be exactly zero and the uon-zero (continous) responses are denoted by ¥1.Ya....¥ig, Where ny + ny =n, There are p regressor variables X).Ny,....X, in the data. Suggest a suitable regression model for predicting the medical expenditure for the (n+ 1)-th individual given the relevant information on the predictors. [5+5] 2. (a) Diseuss the importance of distance correlation in the context of variable selection problem, State three important properties of distance correlation. (hb) Consider atime varying coefficient (UVC) linear regression model for network modeling. How will you test the hypothesis that the fect of the “nearest neighbors” is NOT time-varying? [5+5 3. (a) Define the “systematic component” of a generalized linear model (GLM). What is icant by “canonical link”? For a Binomial GIA, find the canonical link funetion. (b) Let ¥1.¥2, .. Ya be independent random variables from general exponential family with lincar predictor n; = 273 and the link function g. Considering p predictors in the model, how will you use Fisher scoring method to obtain 8, where 6 denotes the MLE of the vector of regression coefficients 8. [4+6] 4. (a) What is the difference between “intermittent missingness” and “monotone missing- ness” for longitudinal responses? Suggest a suitable GLM based approach for imputing the intermittent missing values. ()b) Propose a step by step algorithm for imputing the monotone missing values under the available case missing value (ACMV) restriction. Is this restriction more powerful than the complete-case missing value (CCMY) restriction? Explain (545) (a) In Bayesian framework model parameters are assumed to be random variables with some priot distributions. ‘The regression model with p predictors. response variable (7) and design matrix X can be written as the following: Y'X-A.07 ~ V(XB.@ 1), Suggest suitable priors for # and 0? so that under squared error loss function, the posterior estimates of 3,0? become identical with the respective OLS estimates in the classical approach. Show your work explicitly. (b) How will you get a 95% Bayesian confidence interval for ftom the posterior distribution, (6+4] In a state-space model, observations are collected from 1 subjects at T evenly spaced time points, The observation at time ¢ is modeled as the following: y = uh + e, where ¢'s are iid Normal (0.02). Note that denotes the predictor value at time t. The unknown states 4's are modcled using a first order anto-regressive model as the following: 5, = 8-1 + €:. where q's are iid Normal (0,02). (a) Write down the likelihood function for the above model. (b) Assuming suitable prior distributions for the model parameters, write down the joint posterior distribution. (c) Give a suitable MCMC algorithin for estimating the model parameters. [34-245] INDIAN STATISTICAL INSTITUTE First Semestral Examination : (2015-2016) M. Stat Ist Year Multivariate Analysis Date: 23. 11. 15 Maximum marks: 100 ‘Time: 3 hours. Note: Answer all questions. Maximum you can score is 100, ‘You may use calculators. You can use any result that has been proved in class. However you need to write the results clearly to get credit. Part-A 1. Consider a two-class classification problem in which each component X,,j the measurement vector X is either 0 or 1 with conditional probabilities -+yM of pj = P{X;=1|Class 1} and gj = P{X; = 1 | Class 2}. ‘Assume that the components of X are independent and the prior class probabilities are equal. (a) Find the Bayes classification rule for the problem. 6) (b) If, further pj = p > 3 and g error probability is given by 1 j= 1,...)M, show that for M odd, the Bayes wn 7 y P(M,p)= >> ( Fi \#0-— (9) = (c) Show that Pe(M, p) approaches zero as M — 00. (5) 2. (a) Let X be a p-dimensional random vector. Show that no standardized linear combination (SLO) of X has @ variance larger than that of the first principal component of X. a (b) Show that the variance of any SLC of X which is uncorrelated with the first k principal components of X can not be larger than the variance of the (f+ 1)-th principal component of X. 8] (c) Suppose that XK = (Xi, X2) has a bivariate jmultinomial distribution with n = 1, so that with probability p and Xp=1— X1. Find the principal components of X and their a tl variances. 05) 3. Describe the Principal Pactor Analysis method. P. T. 0] 4, (a) Describe the naive density estimator and express it as a kernel detsity estan 6 (b) Calculate the bias of the naive density estimator f(r) for all real values uf 2 late fay is the uniform(0,1) density and the bandwidth h € (0. 1/3). MW Consider the problem of One-way classification in Multivariate Analysis of Viarmanee Dernse the likelihood ratio test statistic for testing the significance of a fixed contrast 18 Part -B Assignments 1 INDIAN STATISTICAL INSTIUTE Semestral Examination M Stat. (Ist Year) (B — Stream) 2015 — 16 Subject: Statistical Inference Date: 25.11.2015 Full Marks: 60 Duration: 3 hours Attempt all questions (a) Let x,, x2, Xp are iid, Normal (11,7). For fixed a9 > 0, derive likelihood ratio procedure to decide between wo: o < op and wy: 02 0 [Here wand o are unknown] (b) Construct an example to exhibit possible inadmissibility of likelihood ratio test [10 +10) Consider a testing problem where null and alternate hypotheses are composite and have a ‘common boundary. In that context define similar test and test to have Neyman structure with respect to some statistic. Use this to derive UMPU test for one sided hypothesis in context of ‘two population Poisson problem. [24241] {a) Let set of States of Nature be finite and risk set S isa subset of RX. If Sis closed and bounded show that for every prior distribution, Bayes rule exists. (b) fin Problem 3a), S is only closed and bounded below, show by an example that for some prior, Bayes rule may not exist. {10+5] Let in a statistical game, value of the game exists. Show that any minimax rule for the Statistician is an extended Bayes rule. stat bao} INDIAN STATISTICAL INSTITUTE First Semestral Examination: 2015-16 M. Stat. I Year Categorical Data Analysis Date: 27. 11. 15 Full Marks: 100 Time: 3 hours The paper carries 110 points. Answer as many as you can, 1(a) Let Y have a multinomial distribution (n = 1), taking values in k cells with probabilities m,72,....m nepectively. Cosider the measure of diversity defined ny k V(Y) =o m-m). = Show that V(¥) = Pr{¥ # Yo} where ¥; and ¥ are two independent samples from the same multinomial. (b) For the proportional reduction in variation in a I x J table (cell probabilities m,’s etc. in standard notations) with possibly dependent rows (X) and columns (¥), show that 2[V(Y|X)] where V(¥|X) is the measure of diversity defined above conditional of the rows (X) is equal to (0 -D+ D5 nh /ms) (10-415 =25} 2(a) Define generalized linear model with exponential family of distribution for the dependent variable with examples. What is canonical link? For binary data, define a GLM using the log link. Show that effects refer to the relative risk. Give an example when this link is not advisable. (b) Let 7 denote the time to some event, with pdf f and cdf F. For subject i, let t; denote the time to the event and let y = 1 if the event occurs and 0 for censoring (that is, experiment terminated before ¢;). Let T = So; 4 and Y = 3; ys. Assuming f(t) = exp(—X) obtain a simplified expression of the (censored) log likelihood equation and the MLE of 2. (c) Tha hazard function represents the instanteneous rate of the occurance of the event, for subjects who survided upto time ¢ (that is, f(t)/{1 - F(t)] ). In the previous example assume there is a real valued regressor X and subjects are independent with 7; having an exponential distribution with mean [\ exp(B¢;)]~’. Show that the (full) log likelihood (assuming all t,'s are observed). TO. L(,8)= So vi logs — DD ws — Do ulogt where px; = tiexp(Bxy) (10-+15-+20 = 45} 3 (a) Consider logistic regression modeling of a binary Y on real valued regressor X. Suppose for the population Y = j, X has a (43,07) distribution, j = 0,1. Show that Pr{Y = 1|z} satisfies the logistic regression model with 8 = (ua — j10)?/0. (b) IF [X|Y = Jj], j = 0,1 in the above example, are given by N(uj,07) for j = 0,1 respectively, with 09 # 0 show that the logistic regression for Pr{Y = 1|z} still holds (need not be simple linear). [10+15=25| 4. Consider a scenario with J = 3 outcome categories, suppose that expla; + Bj) [iF exp(ar + Aix) + exp(aa + Are)” j=1,2. Show that ms(2) is (i) decreasing in z if B, > 0 and fy >0, (ii) increasing in « if 8; < 0 and 2 <0, (iii) nonmonotonic when 1 and f» have different signs. m3(2) = Midterm Examination Large Sample Statistical Methods Second Semester 2015-2016 Academic Year M.Stat. First Year (B-Stream Only) Date : 22.02.16 Maximum Marks: 40 Duration :- 2} hours Answer as many quyestions as you can. The maximum you can score 40. 1. Suppose Xn ~ N(0,1+3) and Yq = Xql(IXnl 1. Does eX» — e¥» converge in probability ? Justify your answer. B] 2. Suppose X,, ~ Beta(+, +) for all n > 1. Does {X,} converge in distri- bution ? Prove your assertion. 5] 3. Let A, Ag,... be independent events, Prove that a necessary and sufficient condition for P(Us2,A;,) to equal 1 for every subsequence 1< iy < ig <... of integers is : lim inf P(A,) > 0. A] 4, Let X1,-..,Xq be iid with a Poisson (A) distribution with > 0. 1 ‘Are there constants a, and 6, such that a, ((1— 4)"** —,) con- verges in distribution to a non-degenerate random variable, where X, $.X;/n 7 You ate allowed to consider constants a, and/or by which a are dependent on 2. Justify your answer. (5) 5. Let Xi,...,Xp be iid from some distribution F with mean 0 and un- known variance o?. Assume that E(|X|*) < oo for alln > 1. A common test for Ho : 0? = 1 versus Hy: 0? > 1 rejects Ho when $h(X; — Xn)? > xZqen Where $5X,/n and x2,q-1 is the upper & point of a central chisquare distribution with (n— 1) degrees of free- dom, Does Pirg(3:(Xi— Xn)® > x2n-1) converge to a as n-+ 00 if the kurtosis « of the distribution F is non-zero ? Justify your answer. Recall that «= Mf - 3. (6) 6. (a) Let X1, Xo,-.-Xq be iid observations from a N(j1,1) distribution with pp € R unknown. Find the joint asymptotic distribution of (properly centred and scaled) sample mean and sample median. Suppose now a random sample of size 100 has been drawn from & 'N(1,1) distribution and you are only told that the sample median js 1.1. Based on this information, give a reasonable estimate of the sample mean. Justify your answer. (442=6) _.,Xq are iid from a distribution F given by F(x) i) See es —PDIpcesgy: Let Cyn denote the smallest sample median based on X;,--+»Xn- For ” = 10000, how will you aP- proximate the probability that the random variable j,» 8 ree than 0.51 ? Justify your answer. 2] 1 7. Let X1, Xo)... be independent random variables with X, = —ke* with probability e7%*, +-ke* with probability e~**, 1 = —k with probability 5 - e~™*, 1 = +k with probability 5 — et Let Sy = X1+...+Xn. Can you find constants ay, > 0 such that an5q converges to non-degenerate distribution ? Prove your assertion. [5] 8. Let P and Q be two probability measures on (®,.A). Then show that statement (a) below implies statement (b). (a) For any Ac A, {P(A) = 0} implies {Q(A) = 0}. (b) For each ¢ > 0, there exists h, > 0 such that {A € A and P(A) < he} implies {Q(A) < €}. (5 9. Suppose you have a sequence of ii.d observations (X1,¥4), (X2, ¥2),--- from a bivariate distribution G supported on the unit disc {(x,y) : 2? + y® <1}. Suppose further that the distribution has a continuous density 9(2,y) with 9(0,0) > 0. Let D, = VX? + Yf, for i=1,2,.... Let Dan) = min{Di,..., Dn}. Find real constants a, and b, > 0 such that 6, (Dan) ~ %) converges to a non-degenerate distribution. Prove your answer and write the explicit form of the limiting distribution function. (5) INDIAN STATISTICAL InsTITUTE, KOLKATA MIDTERM EXAMINATION: SECOND SEMESTER 2015 - ’16 M.Srar I YEAR Subject : Metric Topology and Complex Analysis ‘Time 2hours 30 minutes Maximum score : 45 Attempt all the problems. Please use a new page to answer each problem and make sure that the problem number in the margin can be read, cven after stapling. If you attempt the same problem several times, please strike out all the attempts except the correct one before submitting your answer script. Justify every step in order to get full credit of your answers. ‘All arguments should be clearly mentioned on the answer script. Points will be deducted for missing or incomplete arguments. (1) Prove that, if is an infinite subset of a compact set KX, then E has @ limit point in K. {9 marks} @) Given a subspace A of a metric space (X, 4), prove that the function +> d(x, A) i uniformly continuous, [8 marks} Suppose (X,d) is a compact metric space and f : X —> X is a function satisfying D a(fle), fy) < day) Vay € Xx # y. Prove that there exists zo € X such that fo [14 marks} ” é (A) Let (Xi) and (¥; da) be two totally bounded metrie spaces Prove that the product . . 2 2 space X xY equipped with the metric d((x1,41), (72,42) aleuea) + dolyr, 2) is again totally bounded. rma W ao a a (f) Decide whether the following pairs of sets are homeomorphic. Justify your answer in x cash case. 1) Rand R° a,)] and (a,b) where a,b € (i) (0,1) and (a,b) where o,6 € R u wie aM ~ W Nya 7A = . ee 1 & ct INDIAN STATISTICAL INSTITUTE Mid-Semeseter of 2nd Semester Examination : 2015-16 Course Name : M.Stat. Ist Year Subject Name Date : Feb 24, 2016 ‘Total Duration 2 : Sample Survey and Design of Experiments 45 mins +45 mins = Note: Use separate answer sheets for two grou _Group - Sample Survey. (Total Marks Answer any three questions. State and prove Godambe's (1955) theorem regarding the existence of uniformly minimum variance estimator for Y within the class of all homogenious linear unbiased estimators. 6) Prove that for a given sample s, if s* denotes the reduced set equivalent to s obtained by ignoring the order and multiplicities of the units appearing in s, and if d* denotes the data al sufficient statistic, corresponding to s*, then d is a mi 6) siven any design p and an unbiased estimator t for ¥ depending on order and/or multiplicity of units in sample s, derive an improved estimator for Y through Rao-Blackwellization, {5) Let P(0-< Py <1, Ef P= 1) be known numbers associated withthe units # of a population aunit i is chosen from U with probability P, and on the ‘second, ae 2B draw a unit j(# #) is chosen with probability 7" U. Suppose on the first draw Fora sample of size 2 drawn under this sampling scheme, write down Des Raj’s (1956) unbiased estimator for Y. Improve that estimator through Rao-Blackwellization INDIAN STATISTICAL INSTITUTE MID-SEMESTRAL EXAMINATION 2016 MSTAT Ist year. Design of Experiments Fobruary 24,2016, Total marks 15 Duration: 45 minutes, Answer all questions. Keep your answers brief and to the point. 1, Suppose you are to weigh 4 objects using a weighing balance with two pans; the balance needing zero bias correction. You are allowed to make 4 observations and suppose all observations are independent with constant variance 0°. a) Derive a lower bound to the variance of the best linear unbiased estimators of these weights. ia) Pao b) Let the d rixbexX =|) ot Uo} p) Let the design matrix = rae r-a-tad Let ¥1. Y2, Ya, ¥4 be the 4 observations, Obtain the estimators of the 4 unknown weights and the variance of the estimators. i) 2. a) Give an example of an experimental situation where you would recommend the use of a block design e} b) Write down a suitable model for analysing data from general block designs. [1] c)Consider the folloving design with 5 treatments labeled 1,2,,...5 and 4 blocks as follows: Block No. | Treatments (i) Check if the following pairwise contrasts are estimable: BI) a) contrast. between | and 2 b) contrast between 3.and 5 ©) contrast between 4 and 5. (ii) Is this design connected? 2 INDIAN STATISTICAL INSTITUTE Mid-Semeseter of 2nd Semester Examination ; 2015-16 Course Name : MStat. Ist Year Subject Name Date : Feb 24, 2016 : Sample Survey and Design of Experiments ‘Total Duration : 45 mins +45 mins = 14 hrs 2 _Note: Use separate answer sheets for two groups. ‘oup ~ Sample Survey. (Total Marks = 15) Answer any three questions. 1, State and prove Godambe's (1955) theorem regarding the existence of uniformly minimum variance estimator for Y within the class of all homogenious linear unbiased estimators. (3) 2, Prove that for a given sample 5, if s* denotes the reduced sct cquivalent to s obtained by ignoring the order and multiplicities of the units appearing in s, and if d* denotes the data corresponding to s*, then d’ is a minimal sufficient statistic. (5) 3. Given any design p and an unbiased estimator ¢ for Y depending on order and/or multiplicity of units in sample 5, derive an improved estimator for ¥ through Rao-Blackwellization (5) A. Let P(0< P;) <1, Dik: Pi = 1) be known numbers associated with the units i of a population it i is chosen from U with probability P, and on the second U.. Suppose on the first draw a uni draw a unit j(¥ #) is chosen with probability asample of size 2 drawn under this sampling scheme, write down Des Ra 7's (1956) unbiased estimator for Y. Improve that estimator through Rao-Blackwellization INDIAN STATISTICAL INSTITUTE MID-SEMESTRAL EXAMINATION 2016 MSTAT Ist year. Design of Experiments February 24, 2016, Total marks 15 Duration: 4 Answer all questions Keep your answers brief and to the point. 1. Suppose you are to weigh 4 objects using « weighing balance with two pans; the balance needing zero bias correction. You are allowed to make 4 observations and suppose all observations are independent with constant variance o. 8) Derive a lower bound to the variance of the best. inear inbiased estimators of these wright i b) Lot the design matic be x= {Eo 1-2 bata Let VY, ¥3% belted observations. Obtain the estizntrsf the unkeown weighs fn the varia ft eatin i 2. a) Give an example of an experimental situation where you would recommend the use Of block desig ti b) White down asitable mode for nasing data fom sen lock designs ¢)Consider the following design with 8 treatments labeled 1,2....5 and 4 blocks as follows: [Block No. [ Treannents -k if the following pairwise contrasts are ostinuble: fa) contrast between 1 and 2. b) contrast between 3. and ) contrast between 4 and 5. Gi) Is this design connected? th INDIAN STATISTICAL INSTITUTE id-Semester Examination; 2015-16 (Second Semester) M. STAT. L YEAR Abstract Algebra Date: 25.02.16 Maxiraurn Marks: 60 Duration: 9} Hours Attempt Question 6 and ANY FOUR from the rest. Q denotes the field of rational numbers, R denotes the field of real numbers and C denotes the field of complex numbers. 1, Let A= R[X]/(X4 — x) (i) Prove that A is isomorphic (as a ring) to the product ring Rx Rx C. (ii) Which clement of R x R x C corresponds to the element X of A? (iii) Describe all prime and maximal ideals of A. (5+2+5=12) 2. Let B=R[X, ¥, Z]/(X? +? + 27-1), (i) Prove that B is a Noetherian integral domain, (ii) Show that X — 1 is a prime element of B. (iii) Examine if B is a PID. (4344219) 3. Let k be a field. (i) Prove that there are infinitely many distinct monic irreducible polynomials in. k[X)) (ii) Deduce that k(X) is not finitely generated as a algebra. [5+7=12] 4, (i) Let R be a UFD with field of fractions K(# R). Prove that K cannot be an algebraically closed field. (ii) Let k be a field. If t is an element in a field extension of k such that t is transcendental over k, then show that the polynomial X” ~ ¢ is irreducible in k(¢)[X]. [6+6=12] 5. (i) Prove that h(X,Y) := X°+3X?¥? +¥44.2X9Y + 3Y? + SY is irreducible in Q[X, ¥} (ii) Let £(X,Y)(F 0) € Q[X,¥]. Show that there exist a,b © Q such that /(a, 6) # 0. [6+6=12) 6. State whether the following statements are TRUE or FALSE with brief justification. Attempt ANY FIVE. (i) IR is any commutative ring and f(X) a non-constant polynomial in R[X] then {(X) has at most finitely many roots in R. (ii) If P is a prime ideal of a commutative ring R and T any ideal of R, then the image of P in R/J is a prime ideal of R/I. (ii) If B is a subring of A such that B and A are isomorphic as rings, then B= A. (iv) IF RC AC R[X,--- Xa] are commutative rings with A being « Noetherian ring, then R must be a Noetherian ting. (jv) If Dlg is a field extension such that {J :k) isa prime number, then l= k(a) for every ae L\k. (4i) If is a field containing C and a is an clement of L such that Cla] = C(a), then « ¢ [3x 5= 15) INDIAN STATISTICAL INSTITUTE Mid-semester Examination (2015-2016) M Stati Optimisation Techniques Date : 25.02.2016 Maximum Marks : 60 Time: 2 hrs. This paper carries 70 marks. Maximum you can score is 60, Precisely justify all your steps. Carefully state all the results you are using, 1. Does there exist 1, 72,73 2 O such that 2 + Se2+a3 = 3 Bry + S82 +ay = Justify your answer. no 2. Consider the following LP problem P: Minimise 2; + 2 subject to xy +22 > 3 Qe tm 2 5 mz. 2 0 (a) Solve the problem graphically. 14] (b) Write down the dual P” (BI (o) Solve P* using Duality Theorem. BI (a) Consider the LP problem P without the constraint 11 2 0 How does that affect the optimal solution? BI (¢) Write the LP problem P inequational form callit Ps ie () Write down the dual Pj. Isthis different from ?"? 1342] 3, Let Abean m xn matrix, ) € R™ where every entry of both A and bare nonnegative and Aisoffull rank, Let c€ B*. Show that the following LP problem has an optimal a solution: {0} Minimise c!'x subject to x 2 Oand Ax 0 and Ax where A is an m x n matrix, € R", 6; € R" for 1,2. Let x; € R" be feasible for P; such that exactly the same coordinates of both zr: and 2 are positive. Show that 21 is optimal for P, if and only if. is optimal for P, (10) Use Simplex method to solve the LP problem and its dual: {20} Maximise ae; + x2 + 2¢y subject to 71,22, 13 > Oand a + % < 3 “21 + 3 < 2 ar + m2 + a <1 Indian Statistical Institute Semester 2 (2015-2016) M, Stat. Ist Year ‘Mid-somester Examination Measure Theoretic Probability Date and Time: 29.2.16,-230 - 430 ‘Total Points: 5 x 6 = 30 Answers must be justified with clear and precise arguments. Ifyou use any theorem/proposition proved in class state it explicitly. All the functions are assumed measurable and the integrals are wrt the appropriate measure. Suppose F is a distribution function with total mass 1 on B and sep is the probability measure on 5(R) obtained through the Caratheodory procedure with sets in the algebra of the form (a,8), Show that the measure of a set A can be approximated from above by open sets and from below by compact sets. (Notice that F may have jumps.) Let (X.F.yp) be a finite measure space and J isa real valued fanetion such that Jj 1% where is postive constant, What can you say about /? 0 = 1,2), Let fa be 8 sequence of nonnegative measurable and integrable functions on (—o0,o0) sith that fu > f cee ee dona that f fo a ff ce, Then show that foreach measurable set B we hve fp fn Jp Let fa bea sequence of functions in L?.1.< p < 00, which converge ae. to a function f€ 27. Show that fa converges to f in L? if [\fallp Ife 5. Let f and g be functions in L4(R) and define f xg to be the function A(y) = [f(y z)o(2)de. For J = LMR) define f by f(s) = fesiOdt. If f,g € LNB) then show that Fg = £9. (tn fact, frst you have to show f+9 € L'(R).) Indian Statistical Institute Semestral Examination Second Semester (2015-2016) MStat. First Year Large Sample Statistical Methods Maximum Marks: 60 Date : 26.04.2016 Duration -- 3} hours Answer all questions Suppose, for each integer n 2 1, Xq denotes a random variable dis- tributed uniformly on the set of points {1/n,2/n,...,1}. Show that, Xn converges in distribution to X where X has a Uniform|0, 1] distri- bution i] Suppose we have non-negative random variables Xp which converge oe surely to X; and E(X) < oo. Show that Xq converges to X in 1 (a) 3, Suppose X1,...,X» are iid Bernoulli(}) random variables. Does there exist a sequence of constants ay such that ax(si — 1/4) converges to 1, S(%i — Xn)? ? Prove your answer and identify the limit if you claim that such a limit exists. [5] a non-degenerate limit, where si 4, Suppose you have two sequences of random variables {Xn} and {Yo} (defined on the same probability space) at least one of which is stochas- tically bounded. Suppose for each t < s, 48 € R, P(Xn < Yn > 8) + 0 as n> co and P(Yn < t.Xn > 8) + 0 as n — 00. Show that Xp — Yq converges to zero in probability. Explain how this fact is useful in deriving the weak Bahadur representation of sample quan tiles. [o+4=10] Suppose X1,...,Xq are iid having a Uniform|# — 1/2,6 + 1/2] distri bution where 6 € Ris unknown. Suppose Xa) < Xa < ++ < Xiw) are the sample order statistics and Xq is the sample mean. Define Yq as follows : Yu = e788 (Xp) — 1/2) + 1 €¥4)(Xqy + 1/2) Js Yq a maximum likelihood estimator of @? Is Yn consistent for @ ? Justify your answers. (B+3= 6. Let X1y..-+Xq be iid with common density f(x, ) given by eee <2<0, ) = eran a feO= Ty eeop where @ € R is unknown. Argue that the maximum likelihood esti- veetor exists and is consistent in this case. Can you give any explicit sequence of functions ga(-) (possibly dependent on m) of Xp such that n(n) (after suitable centering and sealing) converges if distribution. to N(0,1/1(8)), where 1(@) is the Fisher Jnformation based on one observation? Justify your answer. (343-6) 10, Suppose X;,...,Xp are iid with common density f(z,4), where @ € ©, © consisting of only finitely many real numbers. Assume also that density under each @ has the same support and that the distributions under different 6's are different. If pis the true value of @, prove that with probability tending to 1 (under fo) as n —+ 00, the likelihood will be maximized at the value @ = Go 5] |. Let X1,....Xn be iid N(G,1) where @ € R. Let T, denote the Hodges’ estimator that estimates ¢ by 0 if the sample mean lies in [-n-¥/4,n-/4] and by the sample mean otherwise. Derive the asymp- totic distribution of Hodges’ estimator after appropriate centering and scaling. Can you propose a criterion with respect: to which the asymp- totic performance of the sample mean is better than that of the Hodges’ estimator? Justify your answer. (6+2=8] |. Suppose you have an iid sample of size n from a multinomial popula- tion with & classes. Let 7;(> 0), i= 1,2,..., k denote the probabilities i : of the classes, Find the asymptotic distribution of Ty = > Sst7%, where ni,i = 1,2,...,k denote the number of members in the sample falling in the é-th class. If some of the nj's are zero, you may redefine the corresponding term in the sum as zero. (6) Stating appropriate assumptions and the null hypothesis, derive the asymptotic null distribution of the Wilcoxon Signed-Rank Test statis- ties. {6} INDIAN STATISTICAL INSTITUTE M. Stat First Year (2015-16) Second Semester” Resampling Techniques Date: 72/8/2016 Marks: ..19.. Duration: hours, Attempt all questions (1) Let X1,..-.%Xn & F ond ¥,,....Y¥a “ G. Let F and G be absolutely continuous with densities f and g respectively. (i) Develop a bootstrap-based test of the null hypothesis of equality of the medians. (ii) With the given information can you also devise a permutation test for testing median equality? Justify. (iii) If zero is the median for both the distributions with f(0) # 9(0) (both positive), is your bootstrap test asymptotically consistent with the traditional test? Justify. (74+3410=20] (2) Let X1,...,Xn %f F (univariate) with density f. A kernel density esti- raver of js 1< -X A= D*(5*) where «(.) is a given kernel function symmetric about zero, f (z)dx =1, na = f2te(a)de # 0, and \>0 is the bandwidth that determines the smoothness of the estimated kernel density. (j) Propose @ method based on bootstrap and MISE (mean integrated squared error) to determine A (ii) If () is the standard normal density, then compute the bootstrap- based MISE. ; (iii) Show that the bootstrap-based MISE has a bias of the order n-!~* as an estimator of the actual MISE of fa. [3+7+10=20] (3) Consider the following degradation model for studying reliability of system components: m, Q ig = a(t Oc bey FoI wheré-y;; is the measurement of the i-th sample component at time ty; 2(t) = (z1(t), --.,29(¢))” is a g-vector whose components are known func- tions of time t; ; = (Ox,-.-. qi)” are unobservable random q-vectors that are did with a q-variate normal distribution N, (9, Be), where @ and Eq are unknown; ¢; are tid measurement errors with mean zero and vari- ance 1, and also with finite fourth moment; and ©; and e)y are mutually independent. Consider the function R(t) = P(z,(t)@, € Cr; r=1,-.-,9)5 for some appropriate sets Cy,-.., Cg: (i) Obtain a consistent (as n —> 00) estimator R(t) of R(t), (i) Propose, with suitable justifications, the methods of estimating the variance of R(t). Which method do you recommend and why? [10+10=20] (4) Consider the following linear regression model: wet ey i poly where x; is a kx 1 deterministic vector, is the kx 1 vector of parameters, and ¢; are uncorrelated errors with mean zero and unknown variance o? Let X = (x1,...,xq)7 be the design matrix and assume that XX is non-singular. (i) Propose a bootstrap estimator of the covariance matrix of the least squares estimator A of 8. (i) Is your estimator unbiased? Justify. (20-+10=20} (5) (i) In question (4), now propose a jackknife estimator of the covariance matrix of B. Gi) Isit unbiased? Justify (iii) Consider the following weighted jackknife estimator oe 7 a\T pn . tw = Lt wa) (By ~8)" (Bey -B), where ca is the least squares estimator with the i-th observation deleted, and for ij=1, “4 omy wes = (XTX) xs. U condition on y= (RTH) Tey Under what 7 18 Uyy unbiased? (5+5+10=20] Indian Statistical Institute Semester 2, Academic Year: 2015-16 Semestral Examination Course: M, Stat Ist Year Subject: Measure Theoretic Probability Total Points: 5x 14= 70 Date: 2.5.16 ‘Time: 3 Hours Answers must be justified with clear and precise arguments. If you refer to a theorem/result, proved in class, state it explicitly. More than one answer to a question will not be entertained and only the first uncrossed answer will be graded. 1. (a) Let g be a real valued integrable function on a finite measure space such that for some constant M and all simple h, | f ghdy| < Mlinllp, 1

1, X, are independent random variables with the probability distribution 1 X, = 0 with probability 1 ~ — ree = n with probability 1 1 = + with probability +(1 — =) nm alma (a) Does the sequence {Xq} converge with probability 1 or 0? (b) Consider the sum 22, Xe. Does this sum converge with proba- bility 1 or 0? 4, (a) Suppose an’s are positive reals increasing to infinity and 6,’s are reals. Show that 73°(by /an) converges implies (S77 b,)/an converges to zero. (Your argument must use the tails t, = J>>°(bi/a:), which go to zero.) (b) Suppose ¥;’s are independent zero mean random variables with finite variances 1/n. What is the almost sure limit of Li%, Togn 5. Suppose X;, Xp,... are independent random variables such that P(Xq £1) = (1-2) and P(X, = 2%) = 28, k = n+1n+2,.... Let Sn = Xy+++++Xq. Does Sn/ V7 converge weakly to N(0,1)? INDIAN STATISTICAL INSTITUTE SEMESTRAL EXAMINATION MSTAT I YEAR, OPTIMIZATION TECHNIQUES Date:04.05.2016 Maximum marks: 60 Duration: 2 hours The paper contains 67 marks. Answer as much as you can, the maximum you can score is 60. 1. (a) Prove that a graph is bipartite if and only if its incidence matrix is totally unimodular. (8) : (b) Derive a polynomial time algorithm for matching in a bipartite graph. [4] (©) Prove that, in a bipartite graph G, the cardinality of the maximum coclique is equal to the cardinality of the minimum edge cover. {8] (a) Prove that the incidence matrix of a directed graph is totally unimodular. [6] 2. Let ae Zt, and be N. Let K(a,b) @ conv {2 € {0,1)":a"z < b} denote the knapsack polytope. A set. CC {n] is a cover if Dyec a; > 6. (a) Show that the cover inequality jcc; <|C| 11s a valid inequality for K(a,b). [4] (b) Prove that, if C is a cover for K, then the extended cover inequality DS asie-1 EEC) is valid for KC, where E(C)=CU {i : aj 2 4 Vie C}. [A] (c) Consider the knapsack polytope X = { € {0,1)": Ma1H6ry+62yt5zr+5z0-bdzeta7 $ 19}. Find four minimal cover inequalities for X. Find the corresponding extended cover inequalities. (4+4] i nce Problem. Given a sequence S = 3. Consider the Maximum Alternating Sum Subsequence {a1 13,-"",2q) of positive integers, find the subsequence A = {24,)0y/">t, } where in < ip coos < ip, that maximizes the alternating sum y = Zi, ~ Te + + (-DF F2,. You are 2 ed to develop an O(n) dynamic programming algorithm to find the optimal y (NOT the actual subsequence). (a) List the table(s) that your algorithm will use and explain the meaning of each entry. 8 (b) Specify the recurrence and the base case(s) of your algorithm. Argue the correctness. [6] 1 (0) Write a pseudocode for your algorithm. [3] 4, Consider a post office that sell stamps in three different denominations, Rs. 1, Rs. 7, ani} 10. Design a dynamic programming, algorithm that will find the minimurn number of sta, necessary for a postage value of Rs. N’ as well as the denominations. {a) List the table(s) that your algorithm will use and explain the meaning of each entry, (b) Specify the recurrence and the base case(s) of your algorithm. Argue the correctness INDIAN STATISTICAL INSTITUTE Second Semester Examination: 2015-16 M. STAT. I YEAR Abstract Algebra Date: 6.516 Maximum Marks: 70 Duration: 4 Hours Attempt FIVE questions from Group A and THREE from Group B. Q denotes the field of rational numbers. F,, denotes the finite field with q elements, GROUP A Answer ANY FIVE questions. 1. Let L be an algebraic extension of a field . Show that any A-algebra endomorphism of L is an automorphism. Give an example to show that the result would not be true without the hypothesis that L is algebraic over k @ 2. Let J(X) be an irreducible polynomial of degree n over k. Let g(X) € &{X] and A(X) an irreducible factor of /(g(X)) in k[X]. Prove that the degree of A(X) is divisible by n. (Hint: consider field extensions.) 8] 3. Let L be a finite normal extension of k such that no element of L\k is purely inseparable over k. Prove that L is a separable extension of k. is} 4. Let L = F4(t), where tis transcendental over Fy. Let u = 4 +0, K = F4(u) and f(X) = X44°X + u. Show that f is irreducible in K[X] and splits completely in L[X]. Deduce that L is a Galois extension of K of degree 4. [(8+3)+2=8] 5. (i) fais a root of X°+X+1 € Fa[X], then show that a is a primitive 7th root of unity over Fp. (ii) Ifw is another primitive 7th root of unity over Fa, does it necessarily follow that (a) Fa(w) = Fa(a)? (b) wis a root of X8+X +1? (a4(2+2 6. Suppose that the Galois group of an irreducible and separable polynomial f(X) is ‘Abelian. Let E be a splitting field of f(X) over k and let a, .. x be the roots of 7(X) in E. Show that E = k(a;) for each i, 1 R is continuous. (10) 2. Show that a sequence of random variables X,, converges in probability to zero if and only if H(|Xq|/{1+ [Xnl)) + 0 a8 n+ 00 (20) 3. Suppose X1,...,Xn are iid with a N(4, 07) distribution with both pa- rameters being unknown, Find the asymptotic distribution of sample coefficient of variation. (13) 4, State and prove the Weak Bahadur Representation of sample quantiles under iid sampling from a common distribution. (25) 5. Give an example where the maximum likelihood estimator is inconsis- tent. Prove your answer. [19] 6. Stating appropriate assumptions, prove asymptotic normality of se- quences of consistent roots of the likelihood equation (after appropri- ate centering and scaling) (20) 7. Suppose X1,...,Xn are iid having density f(x,6), where @ € R. In- voking appropriate regularity assumptions, derive the asymptotic null distribution of the likelihood ratio test statistic for testing Ho : 8 = 0 versus Hi 10 #0. (19)

You might also like